0 of 15 Questions completed
Questions:
You have already completed the quiz before. Hence you can not start it again.
Quiz is loading…
You must sign in or sign up to start the quiz.
You must first complete the following:
0 of 15 Questions answered correctly
Your time:
Time has elapsed
You have reached 0 of 0 point(s), (0)
Earned Point(s): 0 of 0, (0)
0 Essay(s) Pending (Possible Point(s): 0)
Average score |
|
Your score |
|
An 18-year-old male presents to the ED after having his first generalized tonic-clonic seizure. MRI was unremarkable but a routine EEG showed 4-6 Hz polyspike and wave discharges in sleep. Which of the following antiepileptic drugs are most appropriate to start based on the available information?
A 15-year-old boy with a past medical history of ADHD presents with odd behaviors including shrugging his shoulders and blinking frequently. His mother also states that he used to have some similar movements when he was younger, but they seem to have worsened since starting a new medication for his ADHD. Which of the following medications is most likely responsible for his symptoms?
You are interested in initiating tetrabenazine for a patient with refractory chorea. You should educate the patient that this medication carries a black box warning for which of the following?
Which of the following is the annual conversion rate of mild cognitive impairment (MCI) to dementia?
A 50-year-old male is admitted for 3 months of progressive memory loss, visual hallucinations, and myoclonus. Examination revealed myoclonic jerks. Which of the following tests is the most specific for this disease?
A study published in 1992 indiscriminately assigned 457 patients with optic neuritis to receive either 14 days of oral prednisone, 3 days of intravenous methylprednisolone followed by 11 days of prednisone, or an oral placebo for 14 days as an acute therapy. Visual function testing was performed intermittently over a six-month follow-up period. The results showed that patients who received methylprednisolone followed by oral prednisone had a faster rate of visual recovery. What type of study design was this?
Which of the following medications is the most appropriate pharmacologic agent for paroxysmal kinesigenic dyskinesia?
A 19-year-old male was admitted to the hospital with a 2 weeks history of progressive lower extremity weakness. CSF studies showed an elevated CSF protein and normal WBC count. MRI of the lumbar spine showed contrast enhancement of the cauda equina. His nerve conduction study (NCS) is shown below. What is the most appropriate next step in medical management?
A 65-year-old woman with a past medical history of hypertension presents with recent onset of bilateral eye discomfort and diplopia. She states that she is unable to completely close her eyes. On exam, pain is elicited when testing extraocular movements. Visual acuity and visual fields are unremarkable. MRI of the brain reveals excessive peri-orbital subcutaneous tissue bilaterally. Which of the following is the most likely diagnosis?
Which of the following is most commonly associated with NMDA-receptor encephalitis?
This brain biopsy is representative of which of the following tumors?
Which of the following is the mechanism of action of entacapone?
A 27-year-old woman comes into the clinic with the chief complaint of low energy and depressed mood. Ever since her mother died 3 years ago, she has felt fatigued and has had a poor appetite with a 15-pound weight loss. She denies issues with sleep or concentration and denies feeling guilt or suicidal ideation. Which of the following is the most appropriate diagnosis?
An overweight 54-year-old male presents to the neurology clinic with excessive daytime sleepiness. His wife states that the patient snores excessively during sleep. Which of the following is the most appropriate diagnostic test?
A 65-year-old female presents to the clinic with painless double vision that started abruptly 5 days ago. On exam, pupils are equal, round, and reactive to light bilaterally. On extraocular muscle testing, the left eye has a normal range of motion but the right eye is unable to adduct or elevate. Which of the following is the most likely cause of the patient’s symptoms?